s2Eva_2022PAOI_T1 Comparar integrales numéricos Simpson y Cuadratura de Gauss

Ejercicio: 2Eva_2022PAOI_T1 Comparar integrales numéricos Simpson y Cuadratura de Gauss

Literal a. Integral con Simpson 1/3

Para la ecuación en el intervalo x entre [0,3] aplicando dos veces la fórmula en el intervalo se requieren al menos dos tramos cada Simpson de 1/3. Por lo que la cantidad de tramos es 4 (dos por cada formula, y dos fórmulas) que corresponden a 5 muestras.

El tamaño de paso se calcula como:

h = \frac{b-a}{tramos}=\frac{3-0}{4} = \frac{3}{4} = 0.75

representados en una gráfica como

A = \int_0^3 \frac{e^x \sin(x)}{1+x^2} \delta x

con lo que se define la función del integral f(x)

f(x) = \frac{e^x \sin(x)}{1+x^2}

Con lo que aplicando la fórmula se puede obtener los valores de las muestras:

xi= [0. 0.75   1.5    2.25   3.    ]
fi= [0. 0.9235 1.3755 1.2176 0.2834]

Nota: realizar las expresiones completas para las fórmulas si no adjunta el algoritmo en Python

Aplicando Simpson de 1/3 en cada tramo se tiene:

A_s= \frac{1}{3} \Big( \frac{3}{4} \Big ) \Big( 0 + 4(0.9235) + 1.3755 \Big) + + \frac{1}{3} \Big( \frac{3}{4} \Big ) \Big( 1.3755 + 4(1.2176) +0.2834 \Big) A_s = 2.8998

Literal b. Integral con Cuadratura de Gauss

Para usar dos veces el método de Cuadratura de Gauss se usan dos intervalos, con lo que las muestras en x serán:

xj= [0. 1.5 3. ]

se calculan los valores para el tramo [0, 1.5]:

x_{a1} = \frac{0+1.5}{2} - \frac{1}{\sqrt{3}}\frac{1.5-0}{2} = 0.3169 x_{b1} = \frac{0+1.5}{2} + \frac{1}{\sqrt{3}}\frac{1.5-0}{2} = 1.1830 A_{g1} =\frac{1.5-0}{2} \Big( f(0.3169)+f(1.1830)\Big) = 1.2361

se calculan los valores para el tramo [1.5, 3]:

x_{a2} = \frac{1.5+3}{2} - \frac{1}{\sqrt{3}}\frac{3-1.5}{2} = 1.8169 x_{b2} = \frac{1.5+3}{2} + \frac{1}{\sqrt{3}}\frac{3-1.5}{2} = 2.6830 A_{g2} =\frac{3-1.5}{2} \Big( f(1.8169)+f(2.6830)\Big) = 1.6329

El total del integral para el intervalo  [0,3]

A_g = A_{g1} + A_{g2} = 2.8691

Al comparar los resultados entre los métodos del literal a y b

errado = |A_s - A_g| = 2.8998 - 2.8691 = 0.0307

Instrucciones integradas en Python

# 2Eva_2022PAOI_T1
# Comparar integrales numéricos Simpson
#   y Cuadratura de Gauss
import numpy as np
import matplotlib.pyplot as plt

# INGRESO
fx = lambda x: (np.exp(x)*np.sin(x))/(1+x**2)
a = 0
b = 3

# PROCEDIMIENTO
# Aplicando Simpson 1/3
tramos = 4
muestras = tramos+1
xi = np.linspace(a,b,muestras)
fi = fx(xi)
hs = xi[1]-xi[0]
As = (1/3)*(3/4)*(fi[0]+4*fi[1]+fi[2])
As = As + (1/3)*(3/4)*(fi[2]+4*fi[3]+fi[4])
erradoS = 2*(hs**5)

# Aplicando Cuadratura de Gauss
tramosG = 2
muestrasG = tramosG+1
xj = np.linspace(a,b,muestrasG)
hg = xj[1]-xj[0]

xa1 = (xj[0]+xj[1])/2 - (1/np.sqrt(3))*(xj[1]-xj[0])/2
xb1 = (xj[0]+xj[1])/2 + (1/np.sqrt(3))*(xj[1]-xj[0])/2
Ag1 = (hg/2)*(fx(xa1)+fx(xb1))

xa2 = (xj[1]+xj[2])/2 - (1/np.sqrt(3))*(xj[2]-xj[1])/2
xb2 = (xj[1]+xj[2])/2 + (1/np.sqrt(3))*(xj[2]-xj[1])/2
Ag2 = (hg/2)*(fx(xa2)+fx(xb2))

Ag = Ag1 + Ag2

# error entre métodos
errado = np.abs(As-Ag)

# SALIDA
print('xi=',xi)
print('fi=',fi)
print('A Simpson =', As)
print('error Truncamiento Simpson 2*(h^5):',
      erradoS)
print('Cuadratura de Gauss xa,xb por tramos:',
      [xa1,xb1,xa2,xb2])
print('  fx(xa),fx(xb) por tramos:',
      [fx(xa1),fx(xb1),fx(xa2),fx(xb2)])
print('  integral de cada tramo:', [Ag1,Ag2])
print('A Gauss =', Ag)
print('errado entre Simpson y Gauss',errado)

# Grafica con mejor resolucion
xk = np.linspace(a,b,5*tramos+1)
fk = fx(xk)

plt.plot(xk,fk)
plt.plot(xi,fi,'o')
for unx in xi:
    plt.axvline(unx,color='red',
                linestyle='dotted')
plt.xlabel('x')
plt.ylabel('f(x)')
plt.title('(np.exp(x)*np.sin(x))/(1+x**2)')
plt.grid()
plt.show()

s2Eva_2021PAOII_T2 EDO – Embudos cónicos para llenar botellas

Ejercicio: 2Eva_2021PAOII_T2 EDO – Embudos cónicos para llenar botellas

literal a

La expresión dada en el enunciado para EDO, se reordena para definir la funcion a usar con Runge-Kutta:

\frac{\delta y(t)}{\delta t} + \frac{d^2}{4}\sqrt{2 g \text{ }y(t)}\Bigg[\frac{tan \theta}{y(t)} \Bigg]^2 = 0 \frac{\delta y(t)}{\delta t} = - \frac{d^2}{4}\sqrt{2 g \text{ }y(t)}\Bigg[\frac{tan \theta}{y(t)} \Bigg]^2

siendo h = 0.5,  con y(0) = 0.15 m y d= 0.01 m ajustando las unidades de medida.

\frac{\delta y(t)}{\delta t} = - \frac{0.01^2}{4}\sqrt{2 (9.8) \text{ }y(t)}\Bigg[\frac{tan (\pi/4)}{y(t)} \Bigg]^2 \frac{\delta y(t)}{\delta t} = - (1.1068e-4) \sqrt{ y(t)}\Bigg[\frac{1}{y(t)} \Bigg]^2 \frac{\delta y(t)}{\delta t}= - (1.1068e-4) \frac{\sqrt{ y(t)}}{y(t)^2}

literal b

se inicia el cálculo del siguiente punto de la tabla

i t y
0 0 0.15
1 0.5 0.1490
2 1 0.1480
3 1.5 0.1471

i = 0

K_1 = h\Bigg(- (1.1068e-4) \frac{\sqrt{ y(t)}}{y(t)^2} \Bigg) K_1 = 0.5\Bigg(- (1.1068e-4) \frac{\sqrt{0.15}}{0.15^2}\Bigg) = -9.5258e-04 K_2 = h\Bigg(- (1.1068e-4) \frac{\sqrt{ y(t)+K_1}}{(y(t)+K_1)^2} \Bigg) K_2 = 0.5\Bigg(- (1.1068e-4) \frac{\sqrt{ 0.15+-9.5258e-04}}{(0.15-9.5258e-04)^2} \Bigg) K_2 = -9.6173e-04 y_1 = y_0 + \frac{K_1 + K_2}{2} y_1 = 0.15 + \frac{-9.5258e-04 -9.6173e-04}{2} = 0.149

i = 1

K_1 = 0.5\Bigg(- (1.1068e-4) \frac{\sqrt{0.149}}{0.149^2}\Bigg) =-9.6177e-04 K_2 = 0.5\Bigg(- (1.1068e-4) \frac{\sqrt{ 0.149-9.7120e-04}}{(0.149-9.7120e-04)^2} \Bigg) K_2 = -9.7116e-04 y_2 = y_1 + \frac{-9.6177e-04 + -9.7116e-04}{2} = 0.1480

i = 2

K_1 = 0.5\Bigg(- (1.1068e-4) \frac{\sqrt{0.1480}}{0.1480^2}\Bigg) = -9.7120e-04 K_2 = 0.5\Bigg(- (1.1068e-4) \frac{\sqrt{ 0.1480-9.7120e-04}}{(0.1480-9.7120e-04)^2} \Bigg) K_2= -9.8084e-04 y_3 = y_2 + \frac{-9.7120e-04 + -9.8084e-04}{2} = 0.1471

literal c

Resultados usando Algoritmo, se encuentra que el embudo se vacia entre 3.15 y 3.20 segundos

 [ t , y , K1 , K2 ]
[[ 0.0000e+00  1.5000e-01  0.0000e+00  0.0000e+00]
 [ 5.0000e-01  1.4904e-01 -9.5258e-04 -9.6173e-04]
 [ 1.0000e+00  1.4808e-01 -9.6177e-04 -9.7116e-04]
 [ 1.5000e+00  1.4710e-01 -9.7120e-04 -9.8084e-04]
 [ 2.0000e+00  1.4611e-01 -9.8088e-04 -9.9078e-04]
 [ 2.5000e+00  1.4512e-01 -9.9083e-04 -1.0010e-03]
...
[ 3.1000e+01  2.8617e-02 -7.5631e-03 -1.0583e-02]
 [ 3.1500e+01  1.0620e-02 -1.1431e-02 -2.4563e-02]
 [ 3.2000e+01         nan -5.0566e-02         nan]
 [ 3.2500e+01         nan         nan         nan]

Instrucciones Python

# 2Eva_2021PAOII_T2 EDO – Embudos cónicos para llenar botellas
import numpy as np

def rungekutta2(d1y,x0,y0,h,muestras):
    tamano   = muestras + 1
    estimado = np.zeros(shape=(tamano,4),dtype=float)
    # incluye el punto [x0,y0,K1,K2]
    estimado[0] = [x0,y0,0,0]
    xi = x0
    yi = y0
    for i in range(1,tamano,1):
        K1 = h * d1y(xi,yi)
        K2 = h * d1y(xi+h, yi + K1)

        yi = yi + (K1+K2)/2
        xi = xi + h
        
        estimado[i] = [xi,yi,K1,K2]
    return(estimado)

# INGRESO
d = 0.01
theta = np.pi/4
g = 9.8
d1y = lambda t,y: -(d**2)/4*np.sqrt(2*g*y)*(np.tan(theta)/y)**2

t0 = 0
y0 = 0.15
h  = 0.5
muestras = 70

# PROCEDIMIENTO
tabla = rungekutta2(d1y,t0,y0,h,muestras)

# SALIDA
np.set_printoptions(precision=4)
print('[ t , y , K1 , K2 ]')
print(tabla)

s2Eva_2021PAOII_T1 Promedio de precipitación de lluvia en área

Ejercicio: 2Eva_2021PAOII_T1 Promedio de precipitación de lluvia en área

Los datos de la tabla corresponden a las medidas de precipitación lluviosa en cada cuadrícula. El área observada es de 300×250, los tramos horizontales 6 y los verticales 4.

Los tamaños de paso son:
h = 300/6 = 50
k = 250/4 = 62.5

f(xi,yj)
i \ j 1 2 3 4 5 6
1 0.02 0.36 0.82 0.65 1.7 1.52
2 3.15 3.57 6.25 5 3.88 1.8
3 0.98 0.98 2.4 1.83 0.04 0.01
4 0.4 0.04 0.03 0.03 0.01 0.08

Para obtener el integral doble, usando la forma compuesta de Simpson, primero desarrolla por filas.

i=1

I_1 = \frac{3}{8}(50) ( 0.02+3(0.36)+3(0.82)+0.65) + \frac{1}{3}(50) (0.65+4(1.7)+1.52) = 228.43

i=2

I_2 = \frac{3}{8}(50) ( 3.15+3(3.57)+3(6.25)+5) + \frac{1}{3}(50) (5+4(3.88)+1.8) = 1077.18

i=3

I_3 = \frac{3}{8}(50) ( 0.98+3(0.98)+3(2.4)+1.83) + \frac{1}{3}(50) (1.83+4(0.04)+0.01) = 276.14

i=4

I_4 = \frac{3}{8}(50) ( 0.4+3(0.04)+3(0.03)+0.03) + \frac{1}{3}(50) (0.03+4(0.01)+0.08) = 14.5

con los resultados, luego se desarrolla por columnas:

columnas = [ 228.43,  1077.18,  276.14,  14.5 ]

I_{total} = \frac{3}{8}(62.5) ( 228.43+3(1077.18)+3(276.14)+14.5) I_{total} = 100850.09

para el promedio se divide el integral total para el área de rectángulo.

Precipitacion_{promedio} = \frac{100850.09}{(300)(250)} = 1.34

Solución con algoritmo

Resultados usando algoritmos con Python:

integral por fila: [ 228.4375     1077.1875      276.14583333   14.5       ]
integral total: 100850.09765625
promedio precipitación:  1.34466796875
>>>

Instrucciones con Python

# 2Eva_2021PAOII_T1 Promedio de precipitación de 
#  lluvia en área
import numpy as np

def simpson_compuesto(vector,h):
    m = len(vector)
    suma = 0
    j = 0
    while (j+3)<m:  # Simpson 3/8
        f = vector[j:j+4]
        s = (3*h/8)*(f[0]+3*f[1]+3*f[2]+f[3])
        suma = suma + s
        j = j + 3
    while (j+2)<m: # Simpson 1/3
        f = vector[j:j+3]
        s = (h/3)*(f[0]+4*f[1]+f[2])
        suma = suma + s
        j = j + 2
    while (j+1)<m: # Trapecio
        f = vector[j:j+2]
        s = (h/2)*(f[0]+f[1])
        suma = suma + s
        j = j + 1
    return(suma)
    

# INGRESO
A = [[0.02, 0.36, 0.82, 0.65, 1.7 , 1.52],
     [3.15, 3.57, 6.25, 5.  , 3.88, 1.8 ],
     [0.98, 0.98, 2.4 , 1.83, 0.04, 0.01],
     [0.4 , 0.04, 0.03, 0.03, 0.01, 0.08]]
base = 300
altura = 250

h = base/6
k = altura/4

# PROCEDIMIENTO
A = np.array(A)
[n,m] = np.shape(A)
columna = np.zeros(n,dtype=float)
for i in range(0,n,1):
    vector = A[i]
    integralfila = simpson_compuesto(vector,h)
    columna[i] = integralfila

total = simpson_compuesto(columna,k)
promedio = total/(base*altura)
    
# SALIDA
print('integral por fila:', columna)
print('integral total:', total)
print('promedio precipitación: ', promedio)

s2Eva_2021PAOI_T3 EDP Elíptica con valores en la frontera f(x) g(y)

Ejercicio: 2Eva_2021PAOI_T3 EDP Elíptica con valores en la frontera f(x) g(y)

Dada la EDP elíptica

\frac{\partial ^2 u}{\partial x^2} +\frac{\partial^2 u}{\partial y^2} = 0 0 \lt x \lt \frac{1}{2}, 0 \lt y\lt \frac{1}{2}

Se convierte a la versión discreta usando diferencias divididas centradas:

 


\frac{u[i-1,j]-2u[i,j]+u[i+1,j]}{\Delta x^2} + + \frac{u[i,j-1]-2u[i,j]+u[i,j+1]}{\Delta y^2} = 0

Se agrupan los términos Δx, Δy semejante a formar un λ al multiplicar todo por Δy2

\frac{\Delta y^2}{\Delta x^2}\Big(u[i-1,j]-2u[i,j]+u[i+1,j] \Big) + + \frac{\Delta y^2}{\Delta y^2}\Big(u[i,j-1]-2u[i,j]+u[i,j+1]\Big) = 0

los tamaños de paso en ambos ejes son de igual valor, se simplifica la ecuación

\lambda= \frac{\Delta y^2}{\Delta x^2} = 1
u[i-1,j]-2u[i,j]+u[i+1,j] + + u[i,j-1]-2u[i,j]+u[i,j+1] = 0
u[i-1,j]-4u[i,j]+u[i+1,j] + + u[i,j-1]+u[i,j+1] = 0

que permite plantear las ecuaciones para cada punto en posición [i,j]

En cada iteración se requiere el uso de los valores en la frontera

u(x,0)=0, 0 \leq x \leq \frac{1}{2} u(0,y)=0 , 0\leq y \leq \frac{1}{2} u\Big(x,\frac{1}{2} \Big) = 200 x , 0 \leq x \leq \frac{1}{2} u\Big(\frac{1}{2} ,y \Big) = 200 y , 0 \leq y \leq \frac{1}{2}

Iteraciones

i=1, j=1

u[0,1]-4u[1,1]+u[2,1] + + u[1,0]+u[1,2] = 0

usando los valores en la frontera,

0-4u[1,1]+u[2,1] + 0+u[1,2] = 0 -4u[1,1]+u[2,1] + u[1,2] = 0

i=2, j=1

u[1,1]-4u[2,1]+u[3,1] + + u[2,0]+u[2,2] = 0

usando los valores en la frontera,

u[1,1]-4u[2,1]+200(1/6) + 0+u[2,2] = 0 u[1,1]-4u[2,1] + u[2,2] = -200(1/6)

i=1, j=2

u[0,2]-4u[1,2]+u[2,2] + + u[1,1]+u[1,3] = 0 0 - 4u[1,2]+u[2,2] + u[1,1]+200\frac{1}{6} = 0 - 4u[1,2] + u[2,2]+u[1,1] = -200\frac{1}{6}

i=2, j=2

u[1,2]-4u[2,2]+u[3,2] + + u[2,1]+u[2,3] = 0 u[1,2]-4u[2,2]+200\frac{2}{6} + u[2,1]+200\frac{2}{6} = 0 u[1,2]-4u[2,2]+ u[2,1] = -(2)200\frac{2}{6}

Sistema de ecuaciones a resolver:

\begin{bmatrix} -4 & 1 &1&0\\1 &-4&0&1\\1&0&-4&1 \\0&1&1&-4\end{bmatrix} \begin{bmatrix} u[1,1]\\u[2,1] \\u[1,2]\\u[2,2] \end{bmatrix} = \begin{bmatrix} 0\\-200(1/6)\\-200(1/6)\\-200(4/6) \end{bmatrix}

Resolviendo el sistema se tiene:

[11.11111111 22.22222222 22.22222222 44.44444444]

Instrucciones Python

import numpy as np

A = np.array([[-4, 1, 1, 0],
              [ 1,-4, 0, 1],
              [ 1, 0,-4, 1],
              [ 0, 1, 1,-4]])

B = np.array([0,-200*(1/6),-200*(1/6),-200*(4/6)])

x= np.linalg.solve(A,B)

print(x)

s2Eva_2021PAOI_T1 Masa transportada por tubo

Ejercicio: 2Eva_2021PAOI_T1 Masa transportada por tubo

Las expresiones siguientes se usan dentro de la expresión del integral

Q(t)=9+4 \cos ^2 (0.4t) c(t)=5e^{-0.5t}+2 e^{-0.15 t} M = \int_{t_1}^{t_2} Q(t)c(t) dt

literal a

Usando los valores dados para el intervalo [2,8] con 6 tramos h = (8-2)/6 =1

Se usa los valores de cada ti en Se puede obtener una tabla de valores muestreados para integrar f(t) = Q(i)c(t)

[ti,	 Qi,	 Ci,	 fi]
[ 2.     10.9416  3.321  36.3374]
[ 3.      9.5252  2.3909 22.7739]
[ 4.      9.0034  1.7743 15.9747]
[ 5.      9.6927  1.3552 13.1352]
[ 6.     11.175   1.0621 11.8687]
[ 7.     12.5511  0.8509 10.6793]
[ 8.     12.9864  0.694   9.0121]

Para el integral se usan los valores por cada dos tramos

I\cong \frac{1}{3}[36.3374+4(22.7739) + 15.9747] + \frac{1}{3}[15.9747+4(13.1352) + 11.8687] + \frac{1}{3}[11.8687+4(10.6793) + 9.0121] I = 95.7965

literal b

L acota de error de truncamiento por cada fórmula usada, se estima como O(h5),

error_{trunca} = -\frac{h^5}{90} f^{(4)}(z)

para un valor de z entre [a,b]

por lo que al usar 3 veces la formula de Simpson se podría estimar en:

error_{trunca} = 3(1^5/90) = 0.033

literal c

El resultado se puede mejorar de dos formas:

1. Dado que el número de tramos es múltiplo de 3, se puede cambiar la fórmula a Simpon de 3/8, que tendría una cota de error menor

2. Aumentar el número de tramos disminuyendo el valor de h para que el error disminuya. Por ejemplo si se reduce a 0.5, el error disminuye en el orden de 0.55

Podría recomendar la segunda opión, pues a pesar que se aumenta la cota de error por cada vez que se usa la fórmula, el error de cada una disminuye en ordenes de magnitud 0,03125


La gráfica del ejercicio es:

Instrucciones en Python

import numpy as np
import matplotlib.pyplot as plt

# INGRESO
Q = lambda t: 9+4*(np.cos(0.4*t)**2)
C = lambda t: 5*np.exp(-0.5*t)+2*np.exp(-0.15*t)

t1 = 2
t2 = 8
n  = 6

# PROCEDIMIENTO
muestras = n+1 
dt = (t2-t1)/n
ti = np.arange(t1,t2+dt,dt)
Qi = Q(ti)
Ci = C(ti)
fi = Qi*Ci

# integración con Simpson 1/3
h= dt
I13 = 0
for i in range(0,6,2):
    S13 = (h/3)*(fi[i]+4*fi[i+1]+fi[i+2])
    I13 = I13 + S13

# SALIDA
np.set_printoptions(precision=4)
print("[ti,\t Qi,\t Ci,\t fi]")
for i in range(0,muestras,1):
    print(np.array([ti[i],Qi[i],Ci[i],fi[i]]))
print("Integral S13: ",I13)
# grafica
plt.plot(ti,Qi, label = "Q(t)")
plt.plot(ti,Ci, label = "c(t)")
plt.plot(ti,fi, label = "f(t)")
plt.plot(ti,Qi,'.b')
plt.plot(ti,Ci,'.r')
plt.plot(ti,fi,'.g')
plt.xlabel("t")
plt.ylabel("f(t)=Q(t)*c(t)")
plt.show()

s2Eva_2021PAOI_T2 EDO para cultivo de peces

Ejercicio: 2Eva_2021PAOI_T2 EDO para cultivo de peces

Siendo la captura una constante mas una función periódica,

h(t) = a + b \sin (2 \pi t)

La ecuación EDO del ejercicio, junto a las constantes a=0.9 y b=0.75, r=1

\frac{\delta y(t)}{\delta t} = r y(t)-h(t)

se convierte en:

\frac{\delta y(t)}{\delta t} = (1) y(t)- \Big( 0.9 + .75 \sin (2 \pi t)\Big) \frac{\delta y(t)}{\delta t} = y(t)- 0.9 - .75 \sin (2 \pi t)

Considerando que la población inicial de peces es 1 o 100%, y(0)=1

literal a

h=1/12
tamano = muestras + 1
estimado = np.zeros(shape=(tamano,2),dtype=float)
estimado[0] = [0,1]
ti = 0
yi = 1
for i in range(1,tamano,1):
    K1 = 1/12 * d1y(ti,yi)
    K2 = 1/12 * d1y(ti+1/24, yi + K1/2)
    K3 = 1/12 * d1y(ti+1/24, yi + K2/2)
    K4 = 1/12 * d1y(ti+1/12, yi + K3)

    yi = yi + (1/6)*(K1+2*K2+2*K3 +K4)
    ti = ti + 1/12
        
    estimado[i] = [ti,yi]

literal b

iteración i=0

t(0) = 0

y(0) = 1

K1 = \frac{1}{12} \Big(1- 0.9 - .75 \sin (2 \pi 0)\Big) = 0,008333 K2 = \frac{1}{12} \Big(1- 0.9 - .75 \sin \Big(2 \pi (0+\frac{1}{12})\Big)\Big) = -0.02222 y(1) = 0 + \frac{0.008333+(-0.02222)}{2} = 0.9930 t(1) = 0 + \frac{1}{12} = \frac{1}{12}

iteración i=1

t(1) = \frac{1}{12}

y(1) = 0.9930

K1 = \frac{1}{12} \Big(0.9930 - 0.9 - .75 \sin \Big( 2 \pi\frac{1}{12}\Big)\Big) = -0.02349 K2 = \frac{1}{12} \Big(0.9930 - 0.9 - .75 \sin \Big(2 \pi (\frac{1}{12}+\frac{1}{12})\Big)\Big) = -0.04832 y(1) = 0.9930 + \frac{-0.02349+(-0.04832)}{2} = 0.9571 t(1) = \frac{1}{12} + \frac{1}{12} = \frac{2}{12}

iteración i=2

t(2) = \frac{2}{12}

y(1) = 0.9571

K1 = \frac{1}{12} \Big(0.9571 - 0.9 - .75 \sin \Big( 2 \pi\frac{2}{12}\Big)\Big) = -0.04936 K2 = \frac{1}{12} \Big(0.9571 - 0.9 - .75 \sin \Big(2 \pi (\frac{2}{12}+\frac{1}{12})\Big)\Big) = -0.06185 y(1) = 0.9571 + \frac{-0.04936+(-0.06185)}{2} = 0.9015 t(3) = \frac{2}{12} + \frac{1}{12} = \frac{3}{12}

literal c

Resultado del algoritmo, muestra que la estragegia de cosecha, en el tiempo no es sostenible, dado que la población de peces en el tiempo decrece.

estimado[xi,yi,K1,K2]
[[ 0.0000e+00  1.0000e+00  8.3333e-03 -2.2222e-02]
 [ 8.3333e-02  9.9306e-01 -2.3495e-02 -4.8330e-02]
 [ 1.6667e-01  9.5714e-01 -4.9365e-02 -6.1852e-02]
 [ 2.5000e-01  9.0153e-01 -6.2372e-02 -5.9196e-02]
 [ 3.3333e-01  8.4075e-01 -5.9064e-02 -4.1109e-02]
 [ 4.1667e-01  7.9066e-01 -4.0361e-02 -1.2475e-02]
 [ 5.0000e-01  7.6425e-01 -1.1313e-02  1.8994e-02]
 [ 5.8333e-01  7.6809e-01  2.0257e-02  4.4822e-02]
 [ 6.6667e-01  8.0063e-01  4.5845e-02  5.8039e-02]
 [ 7.5000e-01  8.5257e-01  5.8547e-02  5.5053e-02]
 [ 8.3333e-01  9.0937e-01  5.4907e-02  3.6606e-02]
 [ 9.1667e-01  9.5513e-01  3.5844e-02  7.5807e-03]
 [ 1.0000e+00  9.7684e-01  6.4031e-03 -2.4313e-02]
 [ 1.0833e+00  9.6788e-01 -2.5593e-02 -5.0602e-02]
 [ 1.1667e+00  9.2978e-01 -5.1645e-02 -6.4322e-02]
 [ 1.2500e+00  8.7180e-01 -6.4850e-02 -6.1881e-02]
 [ 1.3333e+00  8.0844e-01 -6.1757e-02 -4.4027e-02]
 [ 1.4167e+00  7.5554e-01 -4.3288e-02 -1.5645e-02]
 [ 1.5000e+00  7.2608e-01 -1.4494e-02  1.5549e-02]
 [ 1.5833e+00  7.2661e-01  1.6800e-02  4.1077e-02]
 [ 1.6667e+00  7.5554e-01  4.2089e-02  5.3969e-02]
 [ 1.7500e+00  8.0357e-01  5.4464e-02  5.0630e-02]
 [ 1.8333e+00  8.5612e-01  5.0470e-02  3.1799e-02]
 [ 1.9167e+00  8.9725e-01  3.1021e-02  2.3563e-03]
 [ 2.0000e+00  9.1394e-01  1.1619e-03 -2.9991e-02]
 [ 2.0833e+00  8.9953e-01 -3.1289e-02 -5.6773e-02]
 [ 2.1667e+00  8.5550e-01 -5.7835e-02 -7.1028e-02]
 [ 2.2500e+00  7.9107e-01 -7.1578e-02 -6.9169e-02]
 [ 2.3333e+00  7.2069e-01 -6.9069e-02 -5.1948e-02]
 [ 2.4167e+00  6.6018e-01 -5.1235e-02 -2.4254e-02]
 [ 2.5000e+00  6.2244e-01 -2.3130e-02  6.1924e-03]
 [ 2.5833e+00  6.1397e-01  7.4142e-03  3.0909e-02]
 [ 2.6667e+00  6.3313e-01  3.1888e-02  4.2918e-02]
 [ 2.7500e+00  6.7053e-01  4.3378e-02  3.8619e-02]
 [ 2.8333e+00  7.1153e-01  3.8421e-02  1.8746e-02]
 [ 2.9167e+00  7.4012e-01  1.7926e-02 -1.1830e-02]
 [ 3.0000e+00  7.4317e-01  0.0000e+00  0.0000e+00]]

Instrucciones en Python

# EDO. Método de RungeKutta 2do Orden 
# estima la solucion para muestras espaciadas h en eje x
# valores iniciales x0,y0
# entrega arreglo [[x,y]]
import numpy as np

def rungekutta2(d1y,x0,y0,h,muestras):
    tamano   = muestras + 1
    estimado = np.zeros(shape=(tamano,4),dtype=float)
    # incluye el punto [x0,y0]
    estimado[0] = [x0,y0,0,0]
    xi = x0
    yi = y0
    for i in range(1,tamano,1):
        K1 = h * d1y(xi,yi)
        K2 = h * d1y(xi+h, yi + K1)

        yi = yi + (K1+K2)/2
        xi = xi + h
        estimado[i-1,2:]=[K1,K2]
        estimado[i] = [xi,yi,0,0]
    return(estimado)

# PROGRAMA PRUEBA
# Ref Rodriguez 9.1.1 p335 ejemplo.
# prueba y'-y-x+(x**2)-1 =0, y(0)=1

# INGRESO
# d1y = y' = f, d2y = y'' = f'
a =0.9; b=0.75; r=1
d1y = lambda t,y: r*y-(a+b*np.sin(2*np.pi*t))
x0 = 0
y0 = 1
h  = 1/12
muestras = 12*3

# PROCEDIMIENTO
puntosRK2 = rungekutta2(d1y,x0,y0,h,muestras)
xi = puntosRK2[:,0]
yiRK2 = puntosRK2[:,1]

# SALIDA
np.set_printoptions(precision=4)
print('estimado[xi,yi,K1,K2]')
print(puntosRK2)


# Gráfica
import matplotlib.pyplot as plt


plt.plot(xi[0],yiRK2[0],
         'o',color='r', label ='[x0,y0]')
plt.plot(xi[1:],yiRK2[1:],
         'o',color='m',
         label ='y Runge-Kutta 2 Orden')

plt.title('EDO: Solución con Runge-Kutta 2do Orden')
plt.xlabel('x')
plt.ylabel('y')
plt.legend()
plt.grid()
plt.show()

s2Eva_IIT2019_T4 Integrar con Cuadratura de Gauss

Ejercicio: 2Eva_IIT2019_T4 Integrar con Cuadratura de Gauss

f(x) = x ln(x)

1 ≤x≤4

se requiere:

I = \int_1^4 x ln(x) dx

literal a. Usando el método de Cuadratura de Gauss con 2 términos

x_a = \frac{b+a}{2} + \frac{b-a}{2}x_0 = \frac{4+1}{2} + \frac{4-1}{2}\Big(\frac{-1}{\sqrt{3}} \Big)

xa =1.6339745962155612

x_b = \frac{b+a}{2} + \frac{b-a}{2}x_1 = \frac{4+1}{2} + \frac{4-1}{2}\Big(\frac{1}{\sqrt{3}} \Big)

xb =3.366025403784439

I \cong \frac{b-a}{2}(f(x_a) + f(x_b)) I \cong \frac{4-1}{2}(x_a ln(x_a) + x_b ln(x_b))

I = 7.33164251999249

literal b.  De la fórmula , despejar el valor del error<0.0001

\Big|\frac{(b-a)}{180}h^4 f^{(4)} (\xi)\Big| <0.0001; \xi \in[a,b] h^4 <0.0001\frac{180}{(4-1)}\frac{1}{f^{(4)} (\xi)} h^4 < 0.006\frac{1}{f^{(4)} (\xi)} h <\Big(0.006\frac{1}{f^{(4)} (\xi)}\Big)^{1/4}

obteniendo la 4ta derivada de la función:

f(x) = x ln(x) f'(x) = ln(x) + x\Big(\frac{1}{x} \Big) = ln(x) +1 f''(x) = \frac{1}{x} f'''(x) = -\frac{1}{x^2} f^{(4)}(x) = 2\frac{1}{x^3}

se tiene que:

h <\Big(0.006\frac{1}{f^{(4)} (\xi)}\Big)^{1/4} h <\Big(0.006\frac{1}{2\frac{1}{\xi^3}}\Big)^{1/4} h <\Big(0.003\xi^3\Big)^{1/4} h <(0.003)^{1/4}\xi^{3/4}

en el peor de los casos, se toma el valor menor de ξ =1

h <(0.003)^{1/4} h<0.2340347319320716

 

s2Eva_IIT2019_T3 EDP elíptica, placa en (1,1)

Ejercicio: 2Eva_IIT2019_T3 EDP elíptica, placa en (1,1)

dada la ecuación del problema:

\frac{\delta ^2 u}{\delta x^2} + \frac{\delta ^2 u}{\delta y^2} = \frac{x}{y} + \frac{y}{x}

1 <  x < 2
1 <  y < 2

Se convierte a la versión discreta usando diferencias divididas centradas:


\frac{u[i-1,j]-2u[i,j]+u[i+1,j]}{\Delta x^2} + + \frac{u[i,j-1]-2u[i,j]+u[i,j+1]}{\Delta y^2} = \frac{x_i}{y_j} + \frac{y_j}{x_i}

Se agrupan los términos Δx, Δy semejante a formar un λ al multiplicar todo por Δy2

\frac{\Delta y^2}{\Delta x^2}\Big(u[i-1,j]-2u[i,j]+u[i+1,j] \Big) + + \frac{\Delta y^2}{\Delta y^2}\Big(u[i,j-1]-2u[i,j]+u[i,j+1]\Big) = =\Delta y^2\Big( \frac{x_i}{y_j} + \frac{y_j}{x_i}\Big)

los tamaños de paso en ambos ejes son de igual valor, se simplifica la ecuación

\lambda= \frac{\Delta y^2}{\Delta x^2} = 1
u[i-1,j]-2u[i,j]+u[i+1,j] + + u[i,j-1]-2u[i,j]+u[i,j+1] = =\Delta y^2\Big( \frac{x_i}{y_j} + \frac{y_j}{x_i}\Big)
u[i-1,j]-4u[i,j]+u[i+1,j] + + u[i,j-1]+u[i,j+1] =\Delta y^2\Big( \frac{x_i}{y_j} + \frac{y_j}{x_i}\Big)

Iteraciones

que permite plantear las ecuaciones para cada punto en posición [i,j]


i=1, j=1

u[0,1]-4u[1,1]+u[2,1] + + u[1,0]+u[1,2] =(0.25)^2\Big( \frac{0.25}{0.25} + \frac{0.25}{0.25}\Big) 0.25ln(0.25)-4u[1,1]+u[2,1] + + 0.25ln(0.25)+u[1,2] = 0.125 -4u[1,1]+u[2,1] +u[1,2] = 0.125 - 2(0.25)ln(0.25) -4u[1,1]+u[2,1] +u[1,2] = 0.8181

i=2, j=1

u[1,1]-4u[2,1]+u[3,1]+ +0.5 ln(0.5)+u[2,2]=0.15625 u[1,1]-4u[2,1]+u[3,1]+u[2,2]=0.15625-0.5 ln(0.5) u[1,1]-4u[2,1]+u[3,1]+u[2,2]=0.8493

i=3, j=1

u[2,1]-4u[3,1]+u[4,1] + + u[3,0]+u[3,2] =(0.25)^2\Big( \frac{0.75}{0.25} + \frac{0.25}{0.75}\Big) u[2,1]-4u[3,1]+u[4,1]+u[3,2] = 0.20833333-0.75 ln(0.75) u[2,1]-4u[3,1]+u[4,1]+u[3,2] =0.4240

Tarea: continuar con el ejercicio hasta plantear todo el sistema de ecuaciones.

A = np.array([
[-4, 1, 0, 1, 0, 0, 0, 0, 0],
[ 1,-4, 1, 0, 1, 0, 0, 0, 0],
[ 0, 1,-4, 0, 0, 1, 0, 0, 0],
[ 1, 0, 0,-4, 1, 0, 1, 0, 0],
[ 0, 1, 0, 1,-4, 1, 0, 1, 0],
[ 0, 0, 1, 0, 1,-4, 0, 0, 1],
[ 0, 0, 0, 1, 0, 0,-4, 1, 0],
[ 0, 0, 0, 0, 1, 0, 1,-4, 1],
[ 0, 0, 0, 0, 0, 1, 0, 1,-4]])
B = np.array(
[0.125 - 2(0.25)ln(0.25),
 0.15625 - 0.5ln(0.5),
 0.20833 - 0.75 ln(0.75),
 ...])

B = [0.8181,
     0.8493,
     0.4240,
     ...]

Algoritmo con Python

Con valores para la matriz solución:

iteraciones:  15
error entre iteraciones:  6.772297286980838e-05
solución para u: 
[[0.        0.2789294 0.6081976 0.9793276 1.3862943]
 [0.2789294 0.6978116 1.1792239 1.7127402 2.2907268]
 [0.6081976 1.1792239 1.8252746 2.5338403 3.2958368]
 [0.9793276 1.7127402 2.5338403 3.4280053 4.3846703]
 [1.3862943 2.2907268 3.2958368 4.3846703 5.5451774]]
>>>

y algoritmo detallado:

# 2Eva_IIT2019_T2 EDO, problema de valor inicial
# método iterativo
import numpy as np

# INGRESO
# longitud en x
a = 1
b = 2
# longitud en y
c = 1
d = 2
# tamaño de paso
dx = 0.25
dy = 0.25
# funciones en los bordes de la placa
abajo     = lambda x,y: x*np.log(x)
arriba    = lambda x,y: x*np.log(4*(x**2))
izquierda = lambda x,y: y*np.log(y)
derecha   = lambda x,y: 2*y*np.log(2*y)
# función de la ecuación
fxy = lambda x,y: x/y + y/x

# control de iteraciones
maxitera = 100
tolera = 0.0001

# PROCEDIMIENTO
# tamaño de la matriz
n = int((b-a)/dx)+1
m = int((d-c)/dy)+1
# vectores con valore de ejes
xi = np.linspace(a,b,n)
yj = np.linspace(c,d,m)
# matriz de puntos muestra
u = np.zeros(shape=(n,m),dtype=float)

# valores en los bordes
u[:,0]   = abajo(xi,yj[0])
u[:,m-1] = arriba(xi,yj[m-1])
u[0,:]   = izquierda(xi[0],yj)
u[n-1,:] = derecha(xi[n-1],yj)

# valores interiores la mitad en intervalo x,
# mitad en intervalo y, para menos iteraciones
mx = int(n/2)
my = int(m/2)
promedio = (u[mx,0]+u[mx,m-1]+u[0,my]+u[n-1,my])/4
u[1:n-1,1:m-1] = promedio

# método iterativo
itera = 0
converge = 0
while not(itera>=maxitera or converge==1):
    itera = itera +1
    # copia u para calcular errores entre iteraciones
    nueva = np.copy(u)
    for i in range(1,n-1):
        for j in range(1,m-1):
            # usar fórmula desarrollada para algoritmo
            fij = (dy**2)*fxy(xi[i],yj[j])
            u[i,j]=(u[i-1,j]+u[i+1,j]+u[i,j-1]+u[i,j+1]-fij)/4
    diferencia = nueva-u
    erroru = np.linalg.norm(np.abs(diferencia))
    if (erroru<tolera):
        converge=1

# SALIDA
print('iteraciones: ',itera)
print('error entre iteraciones: ',erroru)
print('solución para u: ')
print(u)

# Gráfica
import matplotlib.pyplot as plt
from matplotlib import cm
from mpl_toolkits.mplot3d import Axes3D

# matrices de ejes para la gráfica 3D
X, Y = np.meshgrid(xi, yj)
U = np.transpose(u) # ajuste de índices fila es x
figura = plt.figure()

grafica = Axes3D(figura)
grafica.plot_surface(X, Y, U, rstride=1, cstride=1,
                     cmap=cm.Reds)

plt.title('EDP elíptica')
plt.xlabel('x')
plt.ylabel('y')
plt.show()

s2Eva_IIT2019_T2 EDO, problema de valor inicial

Ejercicio: 2Eva_IIT2019_T2 EDO, problema de valor inicial

la ecuación del problema planteado es:

y'(t) = f(t,y) = \frac{y}{2t^3}

0 ≤ t ≤ 1
y(0.5) = 1.5


literal a

La solución empieza usando la Serie de Taylor por ejemplo para tres términos:

y_{i+1} = y_{i} + h y'_i + \frac{h^2}{2!} y''_i x_{i+1} = x_{i} + h E = \frac{h^3}{3!} y'''(z) = O(h^3)

Se observa que se tiene el valor inicial y la primera derivada, si usamos tres términos se puede usar la segunda derivada.

y'(t) = f(t,y) = \frac{y}{2t^3} y''(t) = f'(t,y) = \frac{y'}{2t^3} + y \Big(\frac{-3}{2t^4}\Big) y''(t) = \frac{1}{2t^3}\Big( \frac{y}{2t^3} \Big) - \frac{3y}{2t^4} y''(t) = \frac{y}{4t^6} -\frac{3y}{2t^4}

por lo que la ecuación de Taylor a usar queda de la siguiente forma:

y_{i+1} = y_{i} + h y'_i + \frac{h^2}{2!} y''_i y_{i+1} = y_{i} + h\frac{y}{2t^3}+\frac{h^2}{2} \Big( \frac{y}{4t^6} -\frac{3y}{2t^4}\Big)

que es la ecuacion que se usará con un error de O(h3)

Reemplazando los valores en la fórmula se obtiene la siguiente tabla:

estimado
 [ti,      yi,      d1yi,    d2yi]
[[  0.5    1.5      6.      -12.        ]
 [  0.6    2.04     4.7222  -12.68004115]
 [  0.7    2.4488   3.5697  -10.09510219]
 [  0.8    2.7553   2.6907  -7.46259891]
 [  0.9    2.9870   2.0487  -5.42399041]
 [  1.     3.1648   0.       0.        ]]

cuya gráfica es:


literal b

Para desarrollar Runge-Kutta de 2do orden se dispone de los siguientes datos:

y'(t) = f(t,y) = \frac{y}{2t^3}

t0 = 0.5, y0 = 1.5, h = 0.1

pasos del algoritmo,

K_1 = h * y'(t_i) K_2 = h * y'(t_i+h, y_i + K_1) y_{i+1} = y_i + \frac{K_1+K_2}{2} t_i = t_i + h</p> <p>

iteración 1: i = 0

K_1 = 0.1 * y'(0.5) = 0.6 K_2 = 0.1 * y'(0.5+0.1, 1.5 + 0.6) = 0.4861 y_{1} = 1.5 + \frac{0.6+0.4861}{2} = 2.0430 t_1 = 0.5 + 0.1 = 0.6</p> <p>

iteración 2: i = 1

K_1 = 0.1 * y'(0.6) = 0.4729 K_2 = 0.1 * y'(0.6+0.1, 2.0430 + 0.4729) = 0.3667 y_{1} = 2.0430 + \frac{0.4729+0.3667}{2} = 2.4629 t_1 = 0.6 + 0.1 = 0.7</p> <p>

iteración 3: i = 2

K_1 = 0.1 * y'(0.7) = 0.3590 K_2 = 0.1 * y'(0.7+0.1, 2.4629 + 0.3590) = 0.3667 y_{1} = 2.4629 + \frac{0.3590+0.3667}{2} = 2.7802 t_1 = 0.7 + 0.1 = 0.8</p> <p>

obteniendo la siguiente tabla:

estimado
 [ti,   yi,     K1,     K2]
[[0.5   1.5     0.      0.    ]
 [0.6   2.0430  0.6     0.4861]
 [0.7   2.4629  0.4729  0.3667]
 [0.8   2.7802  0.3590  0.2755]
 [0.9   3.0206  0.2715  0.2093]
 [1.    3.2048  0.2071  0.1613]]
Diferencias entre Taylor y Runge-Kutta2
[ 0.   -0.0030 -0.0140 -0.0248 -0.0335 -0.0400]

Algoritmo en Python

# EDO. Método de Taylor 3 términos
# Runge-Kutta de 2 Orden
# 2Eva_IIT2019_T2 EDO, problema de valor inicial
import numpy as np

# Funciones desarrolladas en clase
def edo_taylor3t(d1y,d2y,x0,y0,h,muestras):
    tamano = muestras + 1
    estimado = np.zeros(shape=(tamano,4),dtype=float)
    # incluye el punto [x0,y0]
    estimado[0] = [x0,y0,0,0]
    x = x0
    y = y0
    for i in range(1,tamano,1):
        estimado[i-1,2:]= [d1y(x,y),d2y(x,y)]
        y = y + h*d1y(x,y) + ((h**2)/2)*d2y(x,y)
        x = x+h
        estimado[i,0:2] = [x,y]
    return(estimado)
def rungekutta2(d1y,x0,y0,h,muestras):
    tamano = muestras + 1
    estimado = np.zeros(shape=(tamano,4),dtype=float)
    # incluye el punto [x0,y0]
    estimado[0] = [x0,y0,0,0]
    xi = x0
    yi = y0
    for i in range(1,tamano,1):
        K1 = h * d1y(xi,yi)
        K2 = h * d1y(xi+h, yi + K1)
        yi = yi + (K1+K2)/2
        xi = xi + h
        estimado[i] = [xi,yi,K1,K2]
    return(estimado)

# PROGRAMA PRUEBA
# INGRESO
# d1y = y', d2y = y''
d1y = lambda t,y: y/(2*t**3)
d2y = lambda t,y: y/(4*t**6)-(3/2)*y/(t**4)
t0 = 0.5
y0 = 1.5
h = 0.1
muestras = 5

# PROCEDIMIENTO
# Edo con Taylor
puntos = edo_taylor3t(d1y,d2y,t0,y0,h,muestras)
ti = puntos[:,0]
yi = puntos[:,1]

# Runge-Kutta
puntosRK2 = rungekutta2(d1y,t0,y0,h,muestras)
# ti = puntosRK2[:,0] # lo mismo del anterior
yiRK2 = puntosRK2[:,1]

# diferencias
diferencia = yi-yiRK2

# SALIDA
print('estimado[ti, yi, d1yi, d2yi]')
print(puntos)

print('estimado[ti, yi, K1, K2]')
print(puntosRK2)

print('Diferencias entre Taylor y Runge-Kutta2')
print(diferencia)

# Gráfica
import matplotlib.pyplot as plt
plt.plot(ti[0],yi[0],'o', color='r',
         label ='[t0,y0]')
plt.plot(ti[1:],yi[1:],'o', color='g',
         label ='y con Taylor 3 términos')
plt.plot(ti[1:],yiRK2[1:],'o', color='m',
         label ='y Runge-Kutta 2 Orden')
plt.title('EDO: Solución numérica')
plt.xlabel('t')
plt.ylabel('y')
plt.legend()
plt.grid()
plt.show())